PT28 LR S1 #20 Forum

Prepare for the LSAT or discuss it with others in this forum.
Post Reply
jim-green

Silver
Posts: 804
Joined: Sat Feb 12, 2011 7:55 pm

PT28 LR S1 #20

Post by jim-green » Wed Apr 06, 2011 3:04 pm

Has anyone done PT28 LR S1 #20? What makes ans A correct?

User avatar
Sh@keNb@ke

Bronze
Posts: 287
Joined: Wed Jul 14, 2010 3:54 am

Re: PT28 LR S1 #20

Post by Sh@keNb@ke » Wed Apr 06, 2011 3:18 pm

jim-green wrote:Has anyone done PT28 LR S1 #20? What makes ans A correct?
W --> PC (Weak --> Prices constant)
W --> UR (Weak --> Unemployment rises)
UR --> ID (Unemployment Rises --> Investment Decreases)

Conclusion: Investment does not decrease.

By negating this you get ~ID --> ~UR --> ~W

(A) States that the economy is weak or investment is decreasing, but we know from our negation that investment is NOT decreasing therefore we can deduct that the economy is NOT weak. (A) is not a possibility and must be false.

jim-green

Silver
Posts: 804
Joined: Sat Feb 12, 2011 7:55 pm

Re: PT28 LR S1 #20

Post by jim-green » Wed Apr 06, 2011 3:41 pm

Sh@keNb@ke wrote:(A) is not a possibility and must be false.
Thanks!

Post Reply

Return to “LSAT Prep and Discussion Forum”